• Shuffle
    Toggle On
    Toggle Off
  • Alphabetize
    Toggle On
    Toggle Off
  • Front First
    Toggle On
    Toggle Off
  • Both Sides
    Toggle On
    Toggle Off
  • Read
    Toggle On
    Toggle Off
Reading...
Front

Card Range To Study

through

image

Play button

image

Play button

image

Progress

1/69

Click to flip

Use LEFT and RIGHT arrow keys to navigate between flashcards;

Use UP and DOWN arrow keys to flip the card;

H to show hint;

A reads text to speech;

69 Cards in this Set

  • Front
  • Back

What to study in Evidence for the MBE:



(know impeachment, character evidence, hearsay, hearsay exceptions - these are the topics tested most heavily)

Exam Tip 1: Make sure you can identify and compartmentalize your character
evidence rules, your impeachment rules, and your hearsay exceptions rules.



Exam Tip 2: When analyzing character evidence, always first determine
whether you are in a criminal or civil case. Different sets of rules apply to
different types of cases.



Exam Tip 3: Memorize the various bases for impeachment and when they are
applicable.



Exam Tip 4: Make sure you can identify whether there is hearsay and the
hearsay exceptions. One exception that is commonly tested is the dying
declaration exception—it is rarely the correct answer!
Examiners love to include
that the declarant knew that death was imminent, but another element is
commonly lacking (for example, the declaration does not have to do about the declarant's death or some other more trivial element).



Exam Tip 5: Watch for fact patterns that combine impeachment, character
evidence, and hearsay exceptions. You’ll often have to examine a fact pattern under two or more theories. Just because it isn't admissible under one theory doesn't mean it isn't admissible under another. If the examiners ask if it's admissible without giving you a theory, they're looking for you to analyze under the various theories.


At a defendant's trial for stealing an automobile, the defendant called a character witness who testified
that the defendant had an excellent reputation for honesty. In rebuttal, the prosecutor calls another
witness to testify that he recently saw the defendant cheat on a college examination.
This evidence should be

A. admitted, because the defendant has "opened the door" to the prosecutor's proof of bad character
evidence.
B. admitted, because the cheating involves "dishonesty or false statement."
C. excluded, because it has no probative value on any issue in the case.
D. excluded, because the defendant's cheating can be inquired into only on cross-examination of the
first witness.


Answer choice D is correct. Although the prosecution cannot introduce evidence of the defendant's
bad character, the defendant makes his character an issue in the case if he offers evidence of his good
character. When the defendant "opens the door," the prosecution is free to rebut the defendant's
claims by attacking the defendant's character. In introducing evidence as to the defendant's character,
the prosecution is generally limited to the same type of evidence that the defendant offered (e.g.,
reputation or opinion); however, on cross-examination, the prosecution may question a character
witness about specific instances of the defendant's conduct. Thus, answer choices A and B are incorrect
because the proper method of impeachment had to be made of the first witness. Answer choice C is
incorrect because such testimony would be relevant, but is excluded because of how the prosecution
sought to introduce it.


A defendant was charged with assault. Upon learning that the defendant intended to testify in his own
defense, the government gave the defense proper notice of its intent to introduce as impeachment
evidence the defendant’s conviction for embezzlement nine years prior. The defense filed a motion to
exclude all evidence of the defendant’s conviction, arguing that it would prejudice the defendant. In a
pre-trial hearing, the judge noted that the conviction would likely have little prejudicial effect.
Is the judge likely to grant the defendant’s motion?

A. No, because the conviction relates to a crime involving dishonesty and occurred within the last 10
years.
B. No, because the probative value of such a conviction outweighs its prejudicial effect.
C. Yes, because a prior conviction may not be used to impeach a defendant who testifies in his own
defense.
D. Yes, because the conviction for embezzlement is not probative in determining whether the
defendant committed an assault

Answer choice A is correct. Any witness, including a criminal defendant, may be impeached with
evidence that he has been convicted of any crime involving dishonesty or false statement within 10
years of conviction. Embezzlement is a crime of dishonesty or false statement. Since the conviction
occurred within the last ten years, the prosecution may use it for impeachment purposes. Answer
choice B is incorrect because it describes the standard used to determine whether the court may bar a
conviction for a crime not involving dishonestly or false statement. A court generally may not exclude
crimes involving dishonesty or false statement that occurred within the prior 10 years. Answer choice C
is incorrect because a conviction for a crime involving dishonesty or false statement within the last 10
years may be used to impeach a criminal defendant who testifies in his own defense. Answer choice D is
incorrect because although the conviction may not be used for the substantive purpose of determining
whether the defendant committed an assault, it may be used to impeach the defendant as a witness.


A patient sued a hospital for medical negligence, claiming that a nurse employed by the hospital failed
to administer critical medication prescribed by the patient's treating physician during the plaintiff's
hospitalization. To prove the nurse's failure to administer the prescribed medication, the patient called
the medical records librarian, who authenticated the hospital's record of the patient's treatment, which
contained no entry showing that the medication in question had been administered.
Is the hospital record admissible?

A. No, because it is hearsay not within any exception.
B. No, because the nurse's testimony would be the best evidence of her actions in treating the
plaintiff.
C. Yes, although hearsay, because it is a statement against interest by agents of the hospital.
D. Yes, because it is within the hearsay exception covering the absence of entries in business records.


Answer choice D is correct. The hospital record itself is hearsay, but it qualifies as a record of regularly
recorded conduct under Federal Rule of Evidence 803(6). The absence of an entry in such a record is
admissible under Rule 803(7) to prove the nonoccurrence of a matter that would normally have been
recorded if it had occurred. Thus, the absence of an entry can be used by the patient to establish that
the medication was not administered. Answer choice A is incorrect. As previously explained, the
hospital record is hearsay, but it is admissible as a record of regularly recorded conduct, and the
absence of an entry in such a record is admissible to prove the nonoccurrence of a matter that would
normally have been recorded if it had occurred. Answer choice B is incorrect, because no rule of
evidence requires the patient to call the nurse to prove whether the medication was administered.
Although the patient has the option of calling the nurse, the patient also has the option of introducing
the hospital record as a business record to prove absence of an entry showing that the medication was
administered. Both forms of proof are permissible. Under the circumstances of this case, where the
nurse may have an incentive to testify falsely, the hospital record is arguably the best evidence. Answer
choice C is incorrect. To fit the statement against interest exception to the hearsay rule, the declarant
must be shown to be unavailable, and there is no such showing here.


A motorcyclist sued a defendant for $50,000 for injuries received when the defendant's truck struck the
motorcyclist at an intersection. The defendant charges the motorcyclist with contributory negligence
and alleges that the motorcyclist failed to have his lights on at a time when it was dark enough to
require them. The defendant offers to have a bystander testify that he was talking to an eyewitness
when he heard the crash and heard the eyewitness, now deceased, exclaim, "That motorcycle doesn't
have any lights on."

The bystander's testimony is

A. admissible as a statement of present sense impression.
B. admissible, because the eyewitness is not available to testify.
C. inadmissible as hearsay, not within any exception.
D. inadmissible, because of the Dead Man Statute.


Answer choice A is correct. A statement describing or explaining an event that is made while the
declarant is perceiving the event (or immediately thereafter) is admissible under the present sense
impression exception to the hearsay rule. In this case, the eyewitness made the statement in question
while perceiving the event, so the statement is admissible. Thus, answer choice C is incorrect. Answer
choice B is incorrect because the statement in this case does not fall under any unavailable declarant
hearsay exceptions (it is not former testimony, it is not a dying declaration, it is not a statement against
interest, it is not a statement about personal or family history, and there is no indication that the
eyewitness died due to any party's wrongdoing). Answer choice D is incorrect because the federal rules
do not have a Dead Man Statute. Even if state law applies in this case, the Dead Man Statue would not
be relevant - Dead Man Statutes apply only when a party with a financial interest in the outcome wants
to testify in a civil case about a communication or transaction with a deceased person whose estate was
party to the suit, which is not the case here.


A defendant was charged with theft of a valuable watch belonging to an elderly resident at the retirement facility where he worked. The defendant argued that he was not a dishonest person. At trial, he sought to introduce the testimony of a resident of the facility that the defendant had found and returned her diamond wedding ring.
Is the woman’s testimony likely to be admitted?


A. No, because a defendant may not introduce evidence of his good character.B. No, because the defendant may not introduce specific instances of conduct to prove character.C. Yes, because a criminal defendant may introduce evidence of his own good character.D. Yes, because the defendant’s character for honesty is relevant to the crime for which he is charged.

Answer choice B is correct. A defendant may introduce evidence of his good character as inconsistent with the type of crime charged. Proof of good character offered by the defendant must be in the form of reputation testimony or opinion testimony. Specific instances of a person’s conduct are not admissible.

A husband was on trial for the murder of his wife. The wife’s death was attributable to a fall which the prosecution alleged was purposefully caused by her husband. Shortly after the wife’s death, the husband told a friend that there was a video recording in which he had stated that his wife’s death “was no accident.” The video recording was found during a proper search of the husband’s office, and is now in police custody. The prosecution, worried that the video recording makes the husband look sympathetic, intends to instead call the friend to testify as to the husband’s statement. The defense has objected to admission of the friend’s testimony.

Should the court permit the friend to testify as to the husband’s statement to him?


A. Yes, because the statement is a statement of a party opponent.B. Yes, because the husband’s statement to his friend is not subject to the original document rule.C. No, because the testimony is hearsay.D. No, because the video recording must be produced.

Answer choice D is correct. The original document rule requires the production of a recording when its contents are at issue. Here, the husband’s statement to his friend is not a direct inculpatory statement, but instead is an admission of making such a statement in a recording. Consequently, the contents of the recording are at issue and the recording itself must be produced since it available. Although there is an exception to the original document rule for an adverse party’s admission as to the contents of a recording, this exception applies only when the admission was made in a deposition, testimony, or written statement.

A defendant was charged with fraud in connection with the sale of nutritional supplements. The prosecution alleged that the defendant verbally represented himself as a physician to convince elderly individuals to sign contracts authorizing the defendant to charge their credit cards monthly fees for deliveries of these supplements. In fact, the defendant had no medical training. The contracts did not identify the defendant as a physician, but the prosecution intended to introduce witness testimony that the defendant verbally represented himself as a physician. The sole issue in dispute is whether the defendant made such representations. At trial, the prosecution introduced a photocopy of a contract between the defendant and one of the alleged victims in order to lay a foundation that the alleged victim bought supplements from the defendant. The defendant did not deny that the alleged victim bought supplements from him, but objected to the introduction of the contract on the grounds that the prosecution was required to introduce an original contract under the best evidence rule. The court sustained the defendant’s objection.

Did the court err in making its ruling?


A. Yes, because the best evidence rule allows for the introduction of an original or duplicate document.B. Yes, because the best evidence rule is not implicated in this case.C. No, because the best evidence rule applies to a document that has a legal effect, such as a contract.D. No, because a photocopy is not admissible when the absence of the original is not explained.

Answer choice B is correct. The best evidence rule requires that the original document or a reliable duplicate be produced to prove the contents of a writing. The rule applies only when the contents of a document are at issue or a witness is relying on the document when testifying. In this case, the contents of the document are not at issue because the sole issue is whether the defendant made verbal representations that he was a physician. Accordingly, the best evidence rule is not implicated in this case. Answer choice A is incorrect because, although it is true that the best evidence rule allows for the introduction of the original or a reliable duplicate, the best evidence rule is not implicated in this case. Answer choice C is incorrect because the best evidence rule applies only when the contents of a document with a legal effect are at issue. The contents of the contract are not at issue in this case. Answer choice D is incorrect because a reliable duplicate may be admissible without any explanation as to why the original was not produced.

A woman was hit by a pizza delivery driver and suffered serious injury. The woman sued the driver for negligence, arguing that he was far exceeding the speed limit in a residential area. She also sued the driver’s employer under a respondeat superior theory. At trial, the woman sought to introduce against the employer evidence that the driver had received five traffic violations during the previous year.
Is the evidence of the driver’s traffic tickets admissible against the employer?


A. No, because the woman may not introduce evidence of the driver’s character trait of bad driving.B. No, because character may not be proved by evidence of prior bad acts in a civil case.C. Yes, because character evidence is generally admissible in civil cases.D. Yes, because the driver’s character is an essential element of the woman’s claim.

Answer choice A is correct. Evidence of a person’s character (or character trait) generally is inadmissible to prove that the person acted in accordance with that character (or character trait) on a particular occasion. In this case, the woman seeks to introduce evidence of the driver’s character trait of bad driving to prove that he was negligent on the occasion in question. Such evidence is inadmissible. Answer choice B is incorrect because, when character evidence is admissible in a civil case, it may be proved by specific instances of conduct. In this case, however, the evidence itself is not admissible. Answer choice C is incorrect because character evidence is generally not admissible in a civil case to prove that a person acted in conformity with that character. Answer choice D is incorrect because the driver’s character is not an essential element of the claim in this case. Character evidence is admissible if it is an essential element of a claim, such as in the case of negligent hiring or entrustment. If the woman had sued the employer under such a theory, the evidence would be admissible. Because the woman sued only under a respondeat superior theory, however, she must prove only that the driver acted negligently, and not that the employer was negligent in hiring the driver or entrusting him with a vehicle. The driver’s character is not an essential element of a negligence claim.


A client sued a lawyer for malpractice for recording the wrong property deed with the county register of deeds. At trial, the attorney representing the client seeks to introduce into evidence a certified copy of the deed that was recorded with the register, who is the proper officer with which to record deeds pursuant to state law. The defendant-lawyer objected to the introduction of the evidence.

Should the judge admit the evidence?


A. No, as it is hearsay not within any exception.B. No, because the original should be presented since the contents are at issue.C. Yes, if a representative of the register of deeds testifies to lay a foundation for admissibility.D. Yes.

Answer choice D is correct. FRE 803(14) establishes a hearsay exception for the record of a document purporting to establish or affect an interest in property, as proof of the content of the original recorded document and its execution and delivery by each person by whom it purports to have been executed, if the record is a record of a public office and an applicable statute authorizes the recording of documents of that kind in that office. Additionally, a copy of an official record or of a document authorized by law to be recorded or filed that was actually recorded or filed in a public office and certified as correct by the custodian or other person authorized to make the certification is self-authenticating. Accordingly, the certified copy of the deed here would be admissible into evidence. Answer choice A is incorrect because there is a hearsay exception for the certified copy of the deed. Answer choice B is incorrect because the best evidence rule generally considers a duplicate (such as a certified copy) an “original” version of the document. Further, the contents of the deed itself are not at issue; what is at issue is whether the defendant-lawyer filed the correct deed with the register of deeds. Answer choice C is incorrect because certified copies of public records are self-authenticating.

A hit man was on trial for murder. His sister, with whom he lived, was called to testify regarding the hit man’s demeanor in the days preceding the killing. She testified that the hit man had been acting erratically and was unusually anxious in the days before the murder. After she testified, the defense attorney called the sister’s ex-husband to the stand. He was prepared to testify that the sister had engaged in several adulterous affairs during their marriage and kept those affairs hidden for many years. The prosecution objected to this testimony.
Should the court allow this testimony?


A. Yes, because the evidence addresses whether the sister is untruthful.B. Yes, because specific instances of conduct are only inadmissible in civil trials.C. No, because the prosecution did not put the sister’s character for truthfulness at issue.D. No, because the ex-husband’s testimony constitutes extrinsic evidence.

Answer choice D is correct. Although a witness’s character for truthfulness or untruthfulness is always relevant, it can only be proved by either opinion or reputation evidence. Extrinsic evidence (including another witness’s testimony) of specific instances of conduct is inadmissible to prove a witness’s character for untruthfulness. Note that character witnesses may be cross-examined about specific instances of conduct, but no extrinsic evidence is permissible. Here, the ex-husband’s testimony about the sister’s secret affairs constitutes extrinsic evidence of specific conduct, and is therefore inadmissible. Answer choice A is incorrect because, while evidence of a witness’s character for truthfulness is relevant and admissible, that evidence must be presented in the form of reputation or opinion evidence, not specific instances of conduct. Answer choice B is incorrect because it misstates the law. Specific instances of conduct are inadmissible in civil cases only in the limited cases in which character evidence is admissible, such as when character is at issue. Answer choice C is incorrect because a witness always puts her credibility at issue by testifying at trial.

A plaintiff sued a defendant in a patent infringement suit. To sustain his claim, the plaintiff was required to demonstrate the formula for a common chemical compound. The plaintiff produced three well-known chemistry texts containing explanations of the formula and asked the court to take judicial notice of the formula for the compound.

Must the trial judge take judicial notice of the formula?


A. No, because a party may not request that the court take judicial notice of a fact, the court may only take judicial notice of a fact on its own.B. No, because the court has discretion not to take judicial notice under these circumstances, if it so chooses.C. Yes, because judicial notice was requested by a party and the court was supplied with the necessary information.D. Yes, but only if the request was made before closing arguments in the case.

Answer choice C is correct. Under FRE 201(d), the court must take judicial notice of an adjudicative fact if a party so requests and provides the court with the necessary information to do so. Here, the fact at issue is an adjudicative fact in that it is not subject to reasonable dispute because it can be accurately and readily determined by resorting to sources whose accuracy cannot reasonably be questioned. The plaintiff made the request and provided the court with three well-known chemistry texts containing the fact. Accordingly, the court was required to take judicial notice of the fact at issue. Answer choice A is incorrect. As noted above, under FRE 201(d), a party may request that the court take judicial notice of a fact. Answer choice B is incorrect because the court here was required to take judicial notice of the fact at issue since a party requested it and provided the court with the necessary information to do so. Answer choice D is incorrect because judicial notice may be taken at any stage of the proceeding, except against a criminal defendant for the first time on appeal.

Prior to trial, a criminal defendant filed a motion to prevent the prosecution from introducing statements by an alleged co-conspirator. At the hearing on the motion, the court definitively ruled that the statements could be admitted into evidence. At the jury trial, the prosecution called the alleged co-conspirator to testify about the statements. The defendant did not object to the introduction of the statements into evidence. On appeal, the defendant claimed that the trial court erred in admitting the statements.

Should the appellate court decide the merits of this claim of error?


A. No, because the defendant failed to object at trial when the prosecution introduced the statements.B. No, because the admissibility of evidence is properly a question for a trial court.C. Yes, because the court made a definitive ruling on the record admitting the statements.D. Yes, because an objection was unnecessary under the plain error rule.

Answer choice C is correct. Once the court makes a definitive ruling on the admissibility of evidence, a party need not renew an objection to the admission of the evidence, even if the ruling was made before the trial began. Answer choice A is incorrect because the defendant was not required to object to the admission of the statements at trial. Answer choice B is incorrect because, although a trial court determines the admissibility of evidence, a party may challenge that ruling on appeal. Answer choice D is incorrect because the plain error rule, which allows a party to appeal after failing to object to the introduction of evidence, does not apply in this case because the defendant did object to the introduction of the statements before trial. Consequently, the appellate court’s review of the admissibility of the statements is not confined to the plain error rule.

In his criminal trial for battery, a defendant sought to admit evidence of his peaceful character. He met the leader of a local animal rights group once during a recent animal rights demonstration in front of the mayor’s office, during which the man succeeded in calming down an angry group of protestors. The defendant planned to ask the group leader to testify about this incident to the jury. The prosecution objected to the introduction of this evidence.
How should the court rule on the objection?


A. Sustain the objection, because this testimony constitutes an inappropriate use of character evidence.B. Sustain the objection, because the leader of the animal rights group only met the defendant once.C. Overrule the objection, because the defendant may introduce evidence of his good character if relevant to the crime charged.D. Overrule the objection, because specific acts are admissible in criminal cases if introduced by the defendant.

Answer choice A is correct. In criminal cases, the defendant is entitled to introduce evidence that his character is inconsistent with the crime charged. However, the only way he can introduce such evidence is through reputation or opinion testimony, not specific acts of conduct. Here, the defendant seeks to introduce evidence of how he acted on one specific occasion; such evidence is inadmissible. Answer choice B is incorrect because if a witness has formed a sufficient opinion of the defendant, or can speak as to the defendant’s reputation in a relevant community, it is irrelevant how many times the witness actually met the defendant. What is more problematic in this situation is that the witness planned to testify about one specific instance of the defendant’s conduct. Answer choice C is incorrect because a criminal defendant is permitted to introduce evidence of his good character as being inconsistent with the type of crime charged. However, not only must that character evidence must be pertinent to the crime charged, it must be in the form of reputation testimony or opinion testimony. Accordingly, answer choice D is also incorrect.

A defendant is convicted of murder and properly sentenced to life imprisonment. Subsequently, the family members of the victim bring a wrongful death action against the defendant. They seek to introduce a properly authenticated, certified copy of the final judgment to show that the defendant wrongfully killed the victim. The defendant objects to the introduction of the judgment.

May the plaintiffs introduce the copy of the final judgment from the defendant’s criminal case?


A. No, because a copy of the judgment does not satisfy the original document rule.B. No, because the judgment is inadmissible hearsay.C. Yes, because the defendant was found guilty of a crime punishable by death or imprisonment of more than one year.D. Yes, because any criminal judgment is admissible in a subsequent civil action.

Answer choice C is correct. The judgment of a conviction in the criminal case is hearsay because it is being introduced to prove that the defendant is responsible for the wrongful death of the victim. Nonetheless, it is admissible under a hearsay exception that permits the admission of such a judgment where the crime is punishable by death or imprisonment of more than one year. Since murder is such a crime, the defendant’s judgment of conviction is admissible under the hearsay exception. Answer choice A is incorrect because the original document rule permits the introduction of a certified copy of a public record. Answer choice B is incorrect because, although the judgment is hearsay, the judgment is admissible under a hearsay exception. Answer choice D is incorrect because only criminal judgments of conviction may be admitted in a subsequent civil action when the conviction was for a crime punishable by death or imprisonment for more than one year.

A defendant was charged and tried for a crime. During the presentation of its case in chief, the prosecution introduced an inculpatory statement made by the defendant in an email regarding the commission of the crime. The defendant requested the immediate introduction of a subsequent related email sent by the defendant that contained an exculpatory statement. The defendant established that fairness required the two statements to be considered at the same time.

Should the court honor the defendant’s request?


A. No, because the statement the defendant sought to introduce was exculpatory.B. No, because the exculpatory statement was not made at the same time as the inculpatory statement.C. Yes, because fairness required the two statements to be considered at the same time.D. Yes, as a matter of judicial notice.

Answer choice C is correct. Under the rule of completeness, a party may compel the introduction into evidence of a writing or recorded statement that in fairness ought to be considered at the same time that related evidence has been introduced by an adverse party. Answer choice A is incorrect because the rule of completeness does not prohibit the introduction of evidence that is adverse to the proponent of the original evidence. In fact, in most cases, the evidence admissible under the rule of completeness will be favorable to the party seeking its admission and adverse to the proponent of the original evidence. Answer choice B is incorrect because the rule of completeness does not require that the writing or recorded statement be a part of the writing or recorded statement introduced by the adverse party. The only requirement is that, in fairness, the two should be considered together. Answer choice D is incorrect because judicial notice permits the court to take notice of a fact that is not subject to reasonable dispute. The evidence for which the defendant seeks immediate admission does not qualify for judicial notice.

In a civil assault suit between a plaintiff and a defendant, a witness testified that the defendant had been with her on the night of the alleged assault, more than 200 miles away from where the assault was alleged to have occurred. To challenge the witness’s credibility, the plaintiff’s attorney sought to present evidence of the witness’s juvenile conviction for voluntary manslaughter five years ago. The defendant objected to the admission of this evidence.

Should the judge admit the evidence?


A. Yes.B. Yes, but only if the judge concludes that the probative value of admitting this evidence outweighs its prejudicial effect to the witness.C. No, but only if the judge concludes that the probative value of admitting this evidence is outweighed by its prejudicial effect to the witness.D. No.

Answer choice D is correct. Under FRE 609(d), evidence of a juvenile conviction is never admissible in a civil case. Accordingly, answer choices A and B are incorrect. Answer choice C is incorrect, as no determination of probative value is necessary. Note that, even in criminal cases, evidence of juvenile convictions has limited admissibility. Evidence of a juvenile conviction may be used to impeach a witness other than the defendant only if (i) it is offered in a criminal case, (ii) an adult’s conviction for that same offense would be admissible to attack the adult’s credibility, and (iii) admitting the evidence is necessary to fairly determine guilt or innocence.

An artist entered into a written agreement to sell a patron a partially finished painting once it was complete. The patron later learned that the artist planned to sell the painting to a third party who offered to pay the artist more than the contract price. The patron filed suit to compel the artist to sell the painting to her in accord with the terms of their agreement, while the artist denied that the painting the artist planned to sell to the third party was the subject of the agreement with the patron. At trial, the patron did not introduce the written agreement or explain its absence. Rather, the patron sought to testify that, when she signed the agreement, the artist had pointed to the painting in question and stated that it was the patron’s painting. The artist’s attorney objected to the testimony that the artist identified the painting as belong to the patron.

How should the court rule?


A. Sustain the objection, because the artist’s statement was hearsay.B. Sustain the objection, because the patron failed to produce the written agreement or explain its absence.C. Overrule the objection, because the statement is not hearsay.D. Overrule the objection, because the statement was relevant.

Answer choice B is correct. Under the best evidence rule, the original document or a reliable duplicate must be used to prove the contents of a writing unless its absence is satisfactorily explained. Here, the plaintiff intends to introduce the statement to prove that the written agreement refers to a specific painting without introducing the written agreement itself. Consequently, the patron may not introduce that statement to establish that the painting in question was the painting that was the subject of the agreement. Answer choice A is incorrect because, although the artist’s statement was being introduced for its truth, it is non-hearsay as a statement of a party to the action. Answer choice C is incorrect because, although the artist’s oral statement was non-hearsay, the best evidence rule prevents its admission into evidence. Answer choice D is incorrect because, while the statement meets the test for relevancy, in that it is both probative and material, it is inadmissible on other grounds.

A defendant was charged with attempted murder. The defendant had previously served a ten-year prison sentence after being convicted of embezzlement in a trial in which the attempted murder victim was the primary witness. The prosecution alleged that, shortly after the defendant was released from prison, the defendant shot at the victim in revenge for the victim’s testimony at the prior trial. Before trial, the prosecution notified the defendant that it intended to introduce evidence that the defendant had been convicted of embezzlement. The defendant, who will not testify at his current trial, objected to the introduction of this evidence regarding his previous conviction. At a pre-trial evidentiary hearing, the judge found that the probative value of the conviction outweighed, but not substantially so, its prejudicial effect.
Is the evidence of the prior conviction likely to be admitted?


A. No, because the probative value of the embezzlement conviction did not substantially outweigh its prejudicial effect.B. No, because the prior crime was committed more than 10 years ago.C. Yes, because the evidence of the defendant’s prior conviction is relevant to show that the defendant acted in conformity with his criminal nature in attempting to murder the witness from his prior trial.D. Yes, because the evidence of the prior conviction demonstrates motive.

Answer choice D is correct. Evidence of a defendant’s crimes or other wrongful acts is not admissible to show his criminal propensity in order to prove that he committed the crime for which he is charged. However, such bad acts are admissible for other purposes, such as proving motive. In this case, the evidence could be used to show that the defendant had motive to shoot the victim. Answer choice A is incorrect because, under Rule 403, evidence may be excluded when its prejudicial effect substantially outweighs its probative value. Here, the probative value instead outweighs the prejudicial effect of the prior conviction. Answer choice B is incorrect because the prior conviction is not being used for impeachment purposes (the defendant will not testify), so the ten-year rule that limits use of a prior conviction for impeachment purposes does not apply. (Note: This rule also does not apply for other reasons, including the fact that the ten-year period is measured from the later of the date of conviction or the date of release from confinement.) Answer choice C is incorrect because, as noted with regard to answer choice D, evidence of a defendant’s other crimes is not admissible to show his criminal propensity in order to prove that he committed the crime for which he is charged.

A testator died with a will that included a bequest to his spouse, “so long [they were] not living separate and apart at the time of [his] death.” The personal representative of the decedent’s estate challenged a claimant’s entitlement to this bequest, arguing that the testator and the claimant were not living together at the time of the testator’s death. The personal representative sought to introduce a sworn statement of a retired judge that, while she was on the bench, she had presided over a dissolution of marriage action between the claimant and the testator. The judge intended to testify that a judgment she issued a week prior to the testator’s death stated, “the parties have sworn that they are currently living separate and apart.” The personal representative had not attempted to obtain a copy of the judgment itself. The claimant objected that this evidence violated the best evidence rule.

Is the court likely to overrule this objection?


A. Yes, because the judge’s statement was made under oath.B. Yes, because the best evidence rule does not apply to matters of personal history.C. No, because proof of a judgment generally requires the introduction of a certified or compared copy of the judgment.D. No, because proof of a judgment through testimony of a witness is not permitted.


Answer choice C is correct. To establish the contents of a public record, such as a judgment, it is generally necessary to introduce a copy of the record that has been certified by the appropriate public custodian. In the alternative, a witness who has compared the copy with the original may testify as to its correctness. Only when neither of these methods is available can the contents of a public record be proved by other evidence. Answer choice A is incorrect because the sworn statement of the judge who issued the judgment constitutes secondary evidence that is not admissible unless the certified or compared copy cannot be obtained by reasonable means. Answer choice B is incorrect because, although most matters of personal history, such as a birth, death or marriage, may be proved through testimonial rather than documentary evidence, a legal judgment requires the issuance of a decree in order to occur. Answer choice D is incorrect because a public record may be established by the introduction of a copy of the record along with the testimony of a person who has compared the copy with the original that the copy is correct. In addition, secondary evidence of a record through testimony of a person with firsthand knowledge of the record can be admissible if neither of the preferred methods is available.


A plaintiff sued a defendant for breach of contract. During the trial, the plaintiff offered into evidence redacted portions of a recorded phone conversation between the parties that tended to show that the defendant breached the contract. The defendant moved to introduce additional portions of the recorded conversation:
Exhibit X, a part of the conversation that suggested that the plaintiff had released the defendant from the disputed contractual obligations;
Exhibit Y, a part of the conversation that dealt with an unrelated business matter between the parties; and
Exhibit Z, a part of the conversation that discussed an ethically questionable business practice employed by the defendant that was not at issue for purposes of the contract. The plaintiff objected to the admission of Exhibits X, Y, and Z, arguing that they were irrelevant and unfairly prejudicial.
How should the court rule?


A. The court should admit Exhibits X, Y, and Z, because of the rule of completeness.B. The court should admit Exhibit X because of the rule of completeness and exclude Exhibits Y and Z because they are irrelevant.C. The court should admit Exhibits X and Y because the exhibits relate to business between the parties, but should exclude Exhibit Z because it would be prejudicial to the defendant.D. The court should exclude Exhibits X, Y, and Z because Exhibit X is unfairly prejudicial and Exhibits Y and Z are irrelevant.

Answer choice B is correct. Under the rule of completeness, when a party introduces part of a writing or recorded statement, an adverse party may compel the introduction of an omitted portion of the writing or statement if, in fairness, it should be considered at the same time, such as when the omitted portion explains or clarifies the admitted portion. The rule of completeness, however, does not require admission of irrelevant portions of a statement. Here, Exhibit X is both probative (tending to make a fact more or less probable than it would be without the evidence) and material (related to the determination of the outcome of the action); therefore, it is relevant and should be admitted. Exhibits Y and Z, which deal with unrelated matters, are irrelevant. Answer choice A is incorrect because the rule of completeness does not require introduction of all of a statement when part of a statement is offered into evidence; irrelevant portions of the conversation may still be excluded. Answer choice C is incorrect because Exhibit Y concerns an unrelated business matter that is neither probative nor material to the current contract dispute between the parties. Answer choice D is incorrect because, although relevant evidence may be excluded if the danger of unfair prejudice substantially outweighs the probative value of the evidence, there is no indication that any unfair prejudice (undue delay, confusion of the issues, misleading the jury) would result from the admission of Exhibit X.

A photographer took a series of pictures of a crowd at a fair. The photographer subsequently learned that a stabbing had taken place at the fair around the time she had taken the pictures. Relying on a detailed physical description of the assailant that had been provided to the police, the photographer examined the photographs and identified the assailant. At the criminal trial of the assailant, the photographer was called as a witness to identify the defendant as the assailant. The defense objected, arguing that the photographer could not testify because the photographs were available.

Should the court rule in the defense’s favor?


A. No, because a photograph is not a writing and therefore is not subject to the original document rule.B. No, because the photographer was present at the scene of the crime.C. Yes, because the photographs are the best evidence of the assailant’s identity.D. Yes, because the trial is criminal, and not civil, in nature.

Answer choice C is correct. The photographer does not have personal knowledge of the identity of the assailant apart from the information she gleaned from examining the photographs. Accordingly, the original document rule, which compels the production of the best evidence where the contents of a writing are at issue, requires that the photographs be produced to prove that the defendant was the assailant, or that their unavailability be explained. Answer choice A is incorrect because the original document rule applies to photographs as well as writings. Answer choice B is incorrect because, although the photographer was present at the scene of the crime, her knowledge as to the identity of the assailant comes from the photographs that she took and from the police description, not from the personal knowledge gained while she was at the crime scene. Answer choice C is incorrect because the original document rule does not distinguish between civil and criminal trials. Consequently, the criminal nature of the trial is irrelevant.

A defendant charged with conspiracy to commit murder requested a meeting with the prosecutor to discuss the defendant’s cooperation. At the beginning of this pre-trial meeting, the prosecutor indicated that she would talk with the defendant only if the defendant agreed that statements made by the defendant during the meeting could be used to impeach the defendant’s testimony if a plea deal could not be reached and the defendant chose to testify at trial. The defendant, after consulting with his attorney, knowingly and voluntarily agreed. No plea agreement was subsequently reached.

At trial, the defendant testified. When the prosecutor attempted to impeach the defendant’s testimony with statements that the defendant had made during the pre-trial meeting, the defendant’s attorney objected.

Should the court permit the prosecutor to ask the defendant about such statements?

Yes. Defendant waived his right not to testify regarding plea bargaining in the presence of his attorney.

A defendant’s father was asked prior to trial about whether his daughter had ever committed violent acts against animals as a child. Her father responded with a nod of his head up and down. He accompanied the nod with the statement, ”Animals are one thing, but it’s heartbreaking to think she could kill a child.” During the defendant’s trial for the murder of her own child, the prosecution sought to introduce into evidence the statement from the defendant’s father, as well as the fact that her father nodded in response to the question, to demonstrate that the defendant had a violent history against animals. The father died immediately prior to the start of the trial.
How should the court rule on these motions to admit the aforementioned evidence?


A. Admit the evidence of the nod only as non-hearsay.B. Admit the evidence of the nod and the statement as non-hearsay.C. Refuse the admission of both the nod and the statement.D. Admit the evidence of the nod and the statement as hearsay falling under an exception.

Answer choice C is correct. A statement is a person’s oral or written assertion, or it may be nonverbal conduct intended as an assertion. Fed. R. Evid. 801(a). An example of assertive conduct is a defendant nodding his head up and down to indicate a “yes” answer to a question. Here, the defendant’s father was deceased and could not testify regarding his words, so the court was using the statements to prove the truth of the matter asserted—that the defendant had actually killed animals and been a violent person. The nod would constitute assertive conduct falling under the definition of hearsay and absent an exception, would be inadmissible. The statement was intended to prove the defendant did kill her own child. Answer choice A is incorrect because the nod is hearsay. Answer choice B incorrect because both the nod and the statement are hearsay. Answer choice D is incorrect because the facts do not indicate that the statement would qualify as an exception to the hearsay rules.

A plaintiff sued a defendant for injuries he suffered while shaving with a razor manufactured by the defendant. The plaintiff alleged that the razor was defectively designed. The plaintiff’s expert witness testified that the manufacturer should have used certain safeguards in the razor’s design that would have made the razor safer. In his testimony, the defendant did not deny that the safeguards urged by the plaintiff’s expert were feasible, but he argued that they were unnecessary and that the razor was not defectively designed. The plaintiff seeks to cross-examine the defendant about a safety modification the defendant has made to the razor since the plaintiff’s injury.

For which of the following substantive purposes may evidence of this modification be used?


A. Both to prove that the razor was defectively designed and to prove that the safeguards described by the expert were feasible.B. Only to prove that the razor was defectively designed.C. Only to prove that the safeguards described by the expert were feasible.D. Neither to prove that the razor was defectively designed nor to prove that the safeguards described by the expert were feasible.

Answer choice D is correct. Evidence of a subsequent remedial measure taken after the plaintiff suffers an injury or harm is not admissible to prove that a product was defectively designed. Although evidence of a remedial measure may be used by a plaintiff to establish that a precautionary measure is feasible, the plaintiff may not introduce such evidence unless the feasibility of such measures is disputed. In this case, the feasibility of the measures is not disputed in this case. Answer choices A, B, and C are incorrect because evidence of subsequent remedial measures may not be used to prove defective design or feasibility of safety modifications when the feasibility off such modifications is not disputed.

A plaintiff brought an action in federal court based on diversity jurisdiction to rescind a contract to transfer real property to the defendant. The plaintiff contended that the defendant’s conduct with regard to the contract constituted duress. Upon the death of the plaintiff, the personal representative of the plaintiff’s estate continued the action.
The applicable state law, which otherwise follows the Federal Rules of Evidence, contains a Dead Man’s statute, which reads:
In any civil proceeding, where any party to a contract in action is dead and his right thereto has passed to a party who represents his interest in the subject in controversy, any surviving party to the contract shall not be a competent witness to any matter occurring before the death of said party.
As part of the case-in-chief, the personal representative introduced an email written by the defendant and sent to the decedent that was relevant to the issue of duress. Immediately thereafter, the defendant, noting that the email was written in response to a letter written by the decedent to the defendant, sought to introduce that letter into evidence. The personal representative objected.

Which of the following is the strongest ground upon which the personal representative can base this objection?


A. The introduction of the letter violates the state’s Dead Man’s statute.B. The letter constitutes inadmissible hearsay.C. The rule of completeness does not apply because the letter was a separate writing.D. Fairness does not require introduction of the letter during the presentation of the personal representative’s case-in-chief.

Answer choice D is correct. The rule of completeness (i.e., Federal Rule 106) permits a party to compel the introduction of a statement that in fairness should be considered at the same time as an admitted writing or recorded statement. Consequently, if fairness does not require the immediate introduction of the prior letter, the defendant will have to wait until the defendant can present evidence in order to introduce this letter. Answer choice A is incorrect. While the Dead Man’s Statute prevents a surviving party to a contract from serving as a witness regarding any matter occurring before the death of another party to the contract, the defendant is not seeking to testify regarding that contract, but instead is seeking to introduce into evidence a written statement made by the decedent. Answer choice B is incorrect because, since the letter was written by the decedent whose estate is the subject of the litigation, the letter would be non-hearsay as a statement by a party-opponent. Answer choice C is incorrect because, while the rule of completeness generally is applied to the introduction of an omitted portion of an admitted writing or recorded statement, this rule can also apply to separate related writing or recorded statement.

A defendant was charged with embezzlement. The prosecution called the defendant’s former employer to testify against the defendant. The defense sought to introduce evidence of the employer’s conviction for felony battery nine years prior. The prosecution filed a motion to exclude the conviction. At a hearing on the motion, the parties presented evidence that the employer had served one month in prison after testifying against his co-defendant regarding their plan to lure a mutual enemy to a dark alley in order to beat him up. At the conclusion of the hearing, the judge noted that the conviction was probative and would have little prejudicial effect.

Is the judge likely to allow the conviction to be admitted to impeach the employer?


A. No, because only a defendant may be impeached by a prior criminal conviction.B. No, because the employer was imprisoned for less than one year.C. Yes, because the employer was convicted of a crime involving dishonesty or false statement.D. Yes, because the probative value of the conviction is not substantially outweighed by its prejudicial effect.

Answer choice D is correct. A witness may be impeached with evidence of a conviction for a felony that is less than 10 years old. If the crime does not involve dishonesty or false statement, the conviction of a witness who is not a criminal defendant will be excluded only if the party objecting to the impeachment shows that its probative value is substantially outweighed by its prejudicial effect. In this case, the judge noted that the conviction was probative and would have little prejudicial effect. Thus, it is likely to be admitted. Answer choice A is incorrect because a witness who is not a criminal defendant may be impeached with evidence of a prior conviction. Answer choice B is incorrect because the employer was convicted of a felony, even though he only served a month of prison time. Answer choice C is incorrect because the crime of battery does not involve dishonesty or false statement, even though the employer acted deceitfully in committing the crime.


A car struck a truck at an intersection. The driver of the truck sued the driver of the car, claiming that the car driver ran a red light. In the investigating officer’s report, the only witness is quoted saying, “I saw the whole thing. The car had the green light.” At trial, the witness testified that he clearly remembered that the car’s traffic light had been red, and that the car ran the light. The defendant did not cross-examine the witness and the witness was dismissed. After the plaintiff had presented his case, the defendant moved to introduce the witness’s statement from the investigating officer’s report solely to impeach the witness’s testimony. The plaintiff objected.

How should the court rule?


A. Overrule the objection, and admit the statement as substantive evidence that the car driver did not run the red light.B. Overrule the objection, and admit the statement as impeachment evidence only.C. Sustain the objection, because the statement is inadmissible hearsay.D. Sustain the objection, because extrinsic evidence may not be used to impeach a witness under the circumstances.

Answer choice D is correct. A witness’s prior statement that is inconsistent with a material part of the witness’s testimony may be used to impeach the witness. However, extrinsic evidence of a witness’s prior inconsistent statement may be introduced only if the witness is given the opportunity to explain or deny the statement and the opposing party is given the opportunity to examine the witness about it. In this case, because the witness was not given an opportunity to explain or deny the statement, the evidence should be excluded. Answer choice A is incorrect because, although the statement is relevant evidence on the substantive issue of whether the car driver did run the red light, the defendant has sought to use the statement only for impeachment purposes (i.e., to call into question the veracity of the witness’s testimony at trial). Answer choice B is incorrect because extrinsic evidence is not admissible for impeachment purposes unless the witness is given the opportunity to explain or deny it. Answer choice C is incorrect because, while the statement would be inadmissible hearsay if it were introduced for its truth, the statement is being introduced for impeachment purposes only.

In a sexual harassment action brought by an employee against her employer, the employee alleged that her supervisor had created a hostile work environment by making repeated crude and explicit sexual comments that were unwelcome. The employer filed a motion to admit evidence that the employee had a sexual relationship with her previous supervisor to show that the employee welcomed the employer’s advances.

How should the court rule on the admissibility of this evidence?


A. The court should refuse to admit evidence of the relationship because evidence offered to prove a victim’s sexual conduct or predisposition is inadmissible in a civil case.B. The court should refuse to admit evidence of the relationship only if its probative value is substantially outweighed by the danger of harm and unfair prejudice to the employee.C. The court should admit evidence of the relationship only if its probative value substantially outweighs the danger of harm and unfair prejudice to the employee.D. The court should admit evidence of the relationship because the exclusion of evidence offered to prove a victim’s sexual conduct or predisposition applies only to a criminal case.

Answer choice C is correct. Evidence offered to prove a victim’s sexual conduct or predisposition is generally not admissible in a civil proceeding involving alleged sexual misconduct, such as a sexual harassment action. Such evidence may be admitted, however, when the court determines at an in camera hearing that the probative value of the evidence substantially outweighs the danger of harm to the victim and unfair prejudice to any party. Answer choice A is incorrect because evidence of a victim’s sexual conduct is admissible if the court determines that the probative value substantially outweighs the danger of harm and unfair prejudice. Answer choice B is incorrect because it misstates the standard. Such evidence is generally not admissible unless the probative value substantially outweighs the danger of harm and unfair prejudice. Answer choice D is incorrect because such evidence is generally not admissible in either civil or criminal cases.

In a probate proceeding, the beneficiary of a purported holographic will sought to introduce a photocopy of that will. An heir of the testator opposed the introduction of this will. Contending that the testator was not of sound mind when the will was written, the heir demanded that the beneficiary produce the original will. The applicable jurisdiction recognizes the validity of a holographic will and has adopted a dead man’s statute.

Is the photocopy of the will admissible without an explanation of the unavailability of the original?


A. Yes, because the photocopy is a duplicate of the original.B. Yes, because the dead man’s statute permits the introduction of the photocopy.C. No, because the will is a document that has legal effect.D. No, because there is question as to the validity of the will.

Answer choice A is correct. The photocopy of the will is a duplicate, since it was produced by a process that accurately produces the original. Consequently, it is admissible to the same extent as the original unless there is a genuine question as to the authenticity of the original or other circumstances that would make its introduction unfair. Since the only challenge advanced by the heir relates to the validity of the will for reasons that lie beyond the authenticity of the original, the photocopy is admissible. Answer choice B is incorrect because a dead man’s statute, does not have a bearing on the admissibility of a photocopy of a testator’s purported will in a probate proceeding. Answer choice C is incorrect because, although the will is a document that has legal effect and thus the original document rule applies, the introduction of a duplicate is permitted. Answer choice D is incorrect because, although the heir has raised a question as to the validity of the will, this question does not go to the authenticity of the original. Consequently, the photocopy of the will is admissible to the same extent as the original.

A defendant was on trial for first-degree murder of his dentist. The prosecution seeks to introduce a properly authenticated recording of the defendant, made two years earlier while negotiating a never-completed plea agreement with the state’s attorney on another charge, in which the defendant stated that he hated his dentist and wished he “would drop off the face of the earth.” The defendant objects to the introduction of the evidence.

Should the judge admit the recording over the defendant’s objection?


A. No, because it is hearsay not within any exception.B. No, because the statement was made while negotiating a plea.C. Yes, because it is being used only to prove bias.D. Yes, because it is the admission of a party opponent.

Answer choice B is correct. FRE 410 provides that statements made in the course of plea discussions with an attorney for the prosecuting authority that do not result in a plea of guilty are not admissible in any civil or criminal proceeding against the defendant who was a participant in the plea discussions. Here, the statement was made in a prior plea negotiation and there was no plea of guilty. Accordingly, the statements would be inadmissible. A

In a trademark infringement action, the plaintiff called its financial manager to testify as to the damages suffered by the plaintiff as a consequence of the defendant’s alleged infringement. The plaintiff sought to introduce a chart prepared by the manager specifically for the litigation. The chart was based on sales reports, which the plaintiff did not introduce into evidence. The sales reports were properly authenticated as business records, but were so voluminous that they could not be conveniently examined in court. The plaintiff made duplicates available for the defendant’s examination and copying. The court overruled the defendant’s objection and admitted the chart into evidence.

Did the court err in doing so?


A. Yes, because the underlying records were not admitted into evidence.B. Yes, because the chart was prepared specifically for the litigation.C. No, because the chart is a business record.D. No, because the documents on which the chart was based were voluminous and could not be conveniently examined in court.

Answer choice D is correct. A chart may be used to prove the contents of voluminous documents that cannot be conveniently examined in court. The proponent of the chart must make either the originals or duplicates available for the other parties to examine and copy, as the plaintiff did in this case. Answer choice A is incorrect because the documents themselves need not be admitted into evidence. Answer choice B is incorrect because, although the chart itself was prepared for litigation and consequently does not qualify as a business record, the chart is admissible because the documents on which it was based could have been admitted as business records. Answer choice C is incorrect because the chart is not a business record because it was prepared for litigation.

A handyman sued a homeowner for negligence after falling off of the homeowner’s roof while engaged in repairing the homeowner’s chimney. The handyman was knocked off of the roof when one of several large dead branches from a nearby tree fell on him. The homeowner’s answer contested that the tree was on the homeowner’s property. At trial, counsel for the handyman seeks to introduce into evidence a properly authenticated invoice, made in the regular course of business, from a landscaping company to the homeowner that indicated that the tree’s dead branches were subsequently trimmed at the homeowner’s expense. The homeowner objected to the admission of the invoice.

Should the judge admit the invoice into evidence over the homeowner’s objection?


A. No, because it is hearsay not within any exception.B. No, because it evidences a subsequent remedial measure.C. Yes, because it is a vicarious admission by the homeowner.D. Yes, because ownership of the tree was disputed.

Answer choice D is correct. Although evidence of subsequent remedial measures is generally not admissible to prove negligence for public policy reasons, it may be admissible for other purposes. One such purpose is to prove ownership or control of the cause of harm, if disputed. Because ownership was disputed here, it is proper for the plaintiff to introduce evidence tending to prove that the defendant owned the tree. Accordingly, answer choice B is incorrect.

A plaintiff sued a forklift manufacturer in a product liability action after a forklift exploded when the propane tank powering the forklift detached from the body of the vehicle. In the action, the parties disputed the make and model of the forklift at issue. Without notice to the defendant forklift manufacturer, and without preliminary authenticating testimony, the plaintiff produced three sets of exhibits for admission into evidence at trial. The exhibits consisted of certified copies of the defendant’s business records identifying the make and model of the unit at issue (Exhibit A); a laser-etched metal plaque that had been affixed to the forklift that identified the make, model, and serial number of the unit (Exhibit B); and a certified copy of the license issued by a state agency identifying the forklift at issue (Exhibit C). The defendant objected to the admission of the exhibits into evidence on the ground that it should have been given written notice before the trial stating the plaintiff’s intent to offer these items into evidence.

How should the judge rule on the defendant’s objection?


A. Sustain the objection as to Exhibit A only.B. Sustain the objection as to Exhibit B only.C. Sustain the objection as to Exhibit C only.D. Sustain the objection as to all three exhibits.

Answer choice A is correct. The court will consider a number of items of evidence to be self-authenticating, meaning that they do not require extrinsic evidence of authenticity in order to be admitted. These include certified copies of public records, trade inscriptions (e.g., labels affixed in the course of business that indicate ownership), and business records. Generally, the proponent of a self-authenticating document is not required to give an adverse party advance notice of the intent to introduce the document. The proponent of a business record must, however, give an adverse party reasonable written notice prior to the trial or hearing of the intent to offer the record and must make the record available for inspection so that the party has a fair opportunity to challenge it. Here, the plaintiff should have sent the defendant notice of its intent to offer business records into evidence to give the defendant time to examine and prepare any appropriate challenges to the records. Answer choice B is incorrect because the metal plaque would be considered a self-authenticating trade inscription which does not require prior written notice before being offered into evidence at a hearing or trial. Answer choice C is incorrect because the signed, sealed public operating license likewise is a self-authenticating document that does not require prior written notice to the adverse party. Answer choice D is incorrect because only the business records in Exhibit A would require prior written notice to the adverse party.


At a defendant’s burglary trial, the defendant testified that the crime was committed not by him, but by an acquaintance whose whereabouts are unknown. The defense seeks to introduce properly authenticated evidence of the acquaintance’s final judgment of conviction by a jury for another burglary, in order to prove that the acquaintance was the type of person who could have committed the burglary for which the defendant is on trial. The prosecution has objected to the introduction of this evidence.

Should the court admit the evidence?


A. No, because the evidence is inadmissible as character evidence.B. No, because the acquaintance is not available to testify.C. Yes, because there is a hearsay exception for a final judgment of conviction.D. Yes, because the defendant may present character evidence.

Answer choice A is correct. Evidence about a person’s character that is used to show that the person acted in conformity with that character trait, including evidence of specific bad acts used to show that person’s propensity to commit a crime, is generally inadmissible. Consequently, the defendant may not introduce evidence that the acquaintance has previously been convicted of the same type of crime to show that the acquaintance committed the crime at issue. Answer choice B is incorrect because a final judgment of conviction is not excluded as hearsay if the judgment was entered after a trial. Accordingly, it is not relevant that the acquaintance is unavailable. Moreover, with respect to the judgment itself, the court or the jury that rendered the judgment (and not the acquaintance) is the declarant. Answer choice C is incorrect because, although there is such an exception to the hearsay rule, this evidence is inadmissible to show the criminal propensity of the acquaintance. Answer choice D is incorrect because, although a defendant may present certain types of character evidence, such as his character for peacefulness when charged with an assault, the defendant is otherwise subject to the rule prohibiting the use of character evidence to show criminal propensity.

A high school teacher played on a hockey team in a local recreational league. During a league game, the teacher was involved in a fight with another hockey player. That player sued the teacher in a battery action to recover for injuries inflicted during the fight. The teacher contended that he had acted in self-defense. The teacher called his principal to testify that the teacher had a reputation within the school community for peacefulness. The plaintiff, who had not introduced evidence of the teacher’s character for violence, objected to this testimony.

Should the court admit this testimony?


A. Yes, because the defendant is entitled to introduce evidence of a pertinent good character trait.B. Yes, because character evidence may be introduced through reputation testimony.C. No, because the plaintiff had not introduced evidence of the teacher’s character for violence.D. No, because such evidence is not admissible in a civil action.

Answer choice D is correct. Evidence of a defendant’s character is inadmissible in a civil case to prove that the defendant acted in conformity with that character trait unless the defendant’s character is an essential element of a claim or defense. Since the defendant’s character for peacefulness is not an element of either battery or self-defense, the principal’s testimony is not admissible. Answer choice A is incorrect because, although a defendant is permitted to introduce evidence of a pertinent good character trait in a criminal case, such evidence is not admissible in a civil case. Answer choice B is incorrect because, although reputation testimony is an acceptable form of presenting character evidence when such evidence is permitted, character evidence is generally not admissible in a civil action. Answer choice C is incorrect because it is not relevant that the plaintiff has not introduced such evidence. Such evidence is not permitted in a civil case, whether introduced by the plaintiff or the defendant.


A 17-year-old male defendant is tried for burglary and vandalism of a local church. The defendant testified on direct examination that he had never been in the church that was vandalized. His counsel asks, "What, if anything, did you tell the police when you were arrested?"

If his answer would be, "I told them I had never been in the church," this answer would be


A. admissible to prove the defendant had never been in the church.B. admissible as a prior consistent statement.C. inadmissible as hearsay not within any exception.D. inadmissible, because it was a self-serving statement by a person with a substantial motive to fabricate.

Answer choice C is correct. While relevant and self-serving, the defendant's statement is still hearsay, because it is an out-of-court statement introduced to prove the truth of the matter asserted. It is immaterial that the statement was made by the defendant himself; it is still hearsay and no exception applies to allow the testimony. Therefore, answer choice A is incorrect. Answer choice B is incorrect because a prior consistent statement is allowed only to rebut an express or implied charge that the witness is lying, and the prior consistent statement must have been made before the declarant had reason to fabricate. In this case, neither of these requirements are satisfied. Answer choice D is incorrect because it is overbroad; not all self-serving statements are inadmissible. The foregoing NCBE MBE question has been modified to reflect current NCBE stylistic approaches; the NCBE has not reviewed or endorsed this modification.

A plaintiff sued a defendant for damages for injuries that the plaintiff incurred when a badly rotted limb fell from a curbside tree in front of the defendant's home and hit the plaintiff. The defendant claimed that the tree was on city property and thus was the responsibility of the city. At trial, the plaintiff offered testimony that a week after the accident, the defendant had cut the tree down with a chainsaw.

The offered evidence is


A. inadmissible, because there is a policy to encourage safety precautions.B. inadmissible, because it is irrelevant to the condition of the tree at the time of the accident.C. admissible to show the tree was on the defendant's property.D. admissible to show the tree was in a rotted condition.

Answer choice C is correct. While evidence of subsequent remedial measures is relevant and can point to an admission of guilt, it is often prohibited through the public policy exception. However, evidence of subsequent remedial measures is admissible to prove ownership or control. Thus, answer choice A is incorrect. Answer choice B is incorrect because the defendant's actions would be relevant, and could indicate the condition of the tree. Answer choice D is incorrect because the evidence is not admissible to show the tree's condition.


A man witnessed a hit-and-run accident in which a truck struck and killed a child. Immediately after the accident, the witness gave police a signed, handwritten statement with a description of the truck, including the make and model, as well as a description of the driver of the truck. After several months, the authorities identified and charged a man who fit the witness’s description, although the pickup truck was never located. In the interim, however, the witness had suffered a brain injury and no longer remembered any of the events from the date of the accident. At trial, the prosecutor called the witness to testify. The prosecutor first showed the witness a photo of a vehicle of the same make and model as the truck in an attempt to refresh the witness’s memory. The defense objected but was overruled by the judge. When this failed to refresh the witness’s memory, the prosecutor showed the witness his earlier handwritten and signed statement. Again, the defense objected and was overruled. The witness testified that he had no memory of the events described in the statement, but that he recognized his handwriting on the statement and that the statement accurately reflected what he witnessed at the time. The prosecutor then moved to introduce the photo and statement into evidence as exhibits, and the defense again objected.

How should the judge rule?


A. The judge should admit both the photo and the statement into evidence as exhibits.B. The judge should admit the photo, but not the statement, into evidence as an exhibit. C. The judge should admit the statement, but not the photo, into evidence as an exhibit.D. The judge should not admit the photo or the statement into evidence as exhibits.

Answer choice D is correct. A witness may examine any item (e.g., writing, photograph) in order to refresh the witness’s present recollection, but the witness’s testimony must be based on the witness’s refreshed recollection, not on the item itself. When the item used to refresh a witness’s recollection is a writing or other record, the adverse party is entitled to have the document produced, to inspect the document, to cross-examine the witness about it, and to introduce any relevant portion into evidence. If a witness is unable to testify about a matter for which a record exists, that record will satisfy the recorded recollection exception to the hearsay rule if: the record is on a matter that the witness once knew about; the record was made or adopted by the witness when the matter was fresh in the witness’s memory; the record accurately reflects the witness’s knowledge; and the witness states that he cannot recall the event well enough to testify fully and accurately, even after consulting the record on the stand. If a record is admitted under this exception, it may be read into evidence, but it may be received as an exhibit only if offered by an adverse party. Here, the prosecutor presented the photos to the witness in an attempt to refresh the witness’s memory and offered the witness’s statement as a recorded recollection. Only the defense would be able to offer the record into evidence. The photo is not admissible because the witness neither authenticated it nor established its relevance. Although the witness’s statement qualifies as a recorded recollection, only the adverse party (here, the defense) may offer it as an exhibit into evidence. Thus, the judge should not admit it based on the prosecutor’s motion. Answer choice A is incorrect because neither the photo nor the statement is admissible as an exhibit. Answer choice B is incorrect because the witness neither authenticated nor established the relevance of the photo. Answer choice C is incorrect because only the adverse party (here, the defense) may offer the statement as an exhibit into evidence.

As in previous years, a band director and an event organizer entered into a written agreement for the director’s band to play at a New Year’s Eve party. Two identical copies of the agreement were prepared, with both the director and organizer signing each copy. The director retained one copy and the organizer retained the other. Prior to the party, the band director threw away his copy of the contract for the upcoming party, mistakenly thinking that it was a contract for a prior year’s party. After the party, the organizer contacted the director and informed him that, since she believed the band had failed to adhere to the terms of the contract, she planned to pay the director only half of the contract amount. The director filed suit to recover the full contractual amount. At trial, the director testified as to why he discarded the contract, and to the terms of the contract. The organizer objected to the testimony regarding the terms of the contract, offering to produce his copy, which the organizer had provided to the director during discovery.

Should the court permit the director to testify as to the terms of the contract?A. No, because the director discarded his copy of the original contract.B. No, because the organizer has a copy of the original contract.C. Yes, because the director was a party to the contract and had personal knowledge of its terms.D. Yes, because the director’s loss of his copy of the original contract was unintentional.

Answer choice B is correct. A written contract, as a document with legal effect, is subject to the best evidence rule. Under this rule, the original must be introduced to prove the terms of the contract unless the unavailability of the original is appropriately explained. Because there were two signed originals, both must be unavailable before the terms of the contract can be proved by other evidence. Since the organizer’s original contract is available, the director cannot prove the terms of the contract through his testimony. Answer choice A is incorrect because, even though the director discarded his original contract, he would be able to testify as to the terms of the contract had that document been the only original. However, since there is a second original that has been made available by the opposing party, the director cannot prove the terms of the contract through his testimony. Answer choice C is incorrect because, although a party may testify as to the contents of a contract or other document that is subject to the best evidence rule when the unavailability of the contract or other document has been satisfactorily explained, the existence of a second original that the opposing party has made available prevents the director from proving the terms of the contract through his testimony. Answer choice D is incorrect because, although the director’s unintentional loss of the contract is a sufficient explanation since the loss was not effected in bad faith (i.e., to prevent the introduction of the original into evidence), the director can prove the terms of the contract through his testimony. The best evidence rule requires that the original possessed by the organizer be introduced to prove the terms of the contract.

A plaintiff sued a defendant for damages for back injuries received in a car wreck. The defendant disputed the damages and sought to prove that the plaintiff's disability, if any, resulted from a childhood horseback riding accident. The plaintiff admitted the childhood accident, but contended it had no lasting effect. The plaintiff calls an orthopedist who had never examined the plaintiff, and poses a hypothetical question as to the cause of the disability that omits any reference to the horseback riding accident. The question was not provided to opposing counsel before trial.

The best ground for objecting to this question would be that


A. the orthopedist lacked firsthand knowledge concerning the plaintiff's condition.B. the hypothetical question omitted a clearly significant fact.C. hypothetical questions are no longer permitted.D. sufficient notice of the hypothetical question was not given to opposing counsel before trial.

Answer choice B is correct. An expert witness may testify as to her opinion provided the witness is qualified as an expert by knowledge, skill, experience, training, or education and the testimony is based on sufficient facts or data. Here, the orthopedist's testimony is useless without the necessary facts of the case (i.e., evidence of the prior accident). Answer choice A is incorrect because an expert does not have to have firsthand knowledge to testify to her opinion; an expert may base her opinion on personal observation, facts made known to the expert at trial, or facts not known to the expert personally but supplied to her outside the courtroom, if they are the type reasonably relied upon by experts in the field. Answer choice C is incorrect because hypotheticals are not required. Answer choice D is incorrect because there is no such requirement.

A plaintiff sued a defendant under a disabilities discrimination statute, alleging that the defendant refused to hire the plaintiff because of her physical handicap. In defense, the defendant asserted that he refused to employ the plaintiff because he reasonably believed that she would be unable to perform the job. The defendant sought to testify that the plaintiff’s former employer advised him not to hire the plaintiff because she was unable to work productively for more than three hours each day.

Is the defendant’s testimony admissible?


A. No, because the defendant’s opinion of the plaintiff’s abilities is not based on personal knowledge.B. No, because the former employer’s statement is hearsay not within any exception.C. Yes, as evidence of the defendant’s reason for refusing to hire the plaintiff.D. Yes, as evidence that the plaintiff would be unable to work longer than three hours each day.

Answer choice C is correct. All evidence that is relevant is admissible unless excluded by a specific rule, law, or constitutional provision. Evidence is deemed relevant if it has a tendency to make a material fact more or less probable than it would be without the evidence. Here, the statement that the former employer made to the defendant is relevant, because it makes his contention that he did not hire the plaintiff because he was concerned that she could not adequately perform more true. Because the evidence is not barred by any specific rule or law, it is admissible. Answer choice A is incorrect. A non-expert witness, such as this defendant, must have personal knowledge of a matter in order to testify about that matter. Here, the defendant does have personal knowledge about the subject of his testimony—his own reasons for deciding not to hire the plaintiff. The defendant is not required to have personal knowledge of the truth of the statement upon which he based those reasons. Answer choice B is incorrect. The former employer’s statement is not being offered to prove the truth of the matter asserted; it is being offered to show that the statement—whether true or not—affected the defendant’s decision not to hire the plaintiff. Accordingly, it does not meet the definition of hearsay. Answer choice D is incorrect. If the defendant’s testimony regarding the former employer’s statement were being offered as evidence that the plaintiff would be unable to work longer than three hours each day, it would constitute inadmissible hearsay, as it would be introducing an out-of-court statement for the purpose of proving the truth of that statement.

In a civil action being tried before a jury, a party objected to the introduction of evidence on the grounds that disclosure of the evidence was protected by the psychotherapist-patient privilege. In ruling on this objection, the court considered evidence protected by the attorney-client privilege.

Has the court acted properly in making its ruling?


A. No, because the court considered privileged evidence in making it ruling.B. No, because the issue of the existence of a privilege is a matter for the jury, not the court to decide.C. Yes, because the court is not bound by the rules of evidence when making determinations as to admissibility.D. Yes, because the action was a civil action, not a criminal action.

Answer choice A is correct. In determining a preliminary question, such as the existence of a privilege, although the court is not generally bound by the rules of evidence, the court cannot consider privileged evidence. Answer choice B is incorrect because the existence of a privilege is a matter for the court, not the jury to decide. Answer choice C is incorrect because, while the court is not generally bound by the rules of evidence in making preliminary determinations, it cannot consider privileged evidence in making its determination. Answer choice D is incorrect because the rule allocating preliminary questions to the court rather than the jury applies to both criminal as well as civil action.

During a defendant’s trial for the murder of her husband, the prosecution attempted to submit two pieces of evidence to establish that she committed the crime. The evidence included a printout of a phone tracking device indicating that the husband’s mobile phone had called 911 a minute before his death, and a written statement from the 911 operator to her supervisor, made at the supervisor’s request after charges against the defendant were filed, indicating that the operator did not speak to the defendant’s husband. The defendant did not object to the relevance of the evidence but did object that both pieces of evidence were hearsay not falling under any exception; the prosecution argued that the evidence was not hearsay at all.
How should the court rule on the admission of this evidence?


A. Allow both the printout and the written statement.B. Allow the printout only.C. Allow the statement from the phone operator only.D. Prohibit both the printout and the written statement.

Answer choice B is correct. Hearsay is a statement that the declarant makes at a time other than while testifying at the current trial or hearing (i.e., an out-of-court statement) that is offered to prove the truth of the matter asserted. Hearsay evidence generally is inadmissible unless it falls within an exception. Evidence generated by a machine, such as an automatically generated time stamp on a fax, a printout of results of computerized telephone tracing equipment, and raw data (such as blood-alcohol level) generated by a forensic lab’s diagnostic machine, is not hearsay. Accordingly, the printout is not a hearsay statement and will not be barred from admissibility for the reason of hearsay. However, the statement from the operator to her supervisor does not appear to fall within any hearsay exception. Although records kept in the ordinary course of business may be admissible as an exception to the hearsay rule, there is nothing in the fact pattern indicating that this statement was made in the ordinary course of business. In fact, the facts seem to indicate that the statement was made specifically for the purposes of litigation, which would take the statement outside the hearsay exception. Answer choice A is incorrect because, although the printout is not hearsay, the written statement is hearsay not falling under any exception. Answer choice C is incorrect because the statement from the phone operator is hearsay not falling under an exception. Answer choice D is incorrect because the printout is allowed as nonhearsay.

The defendant and a friend were charged with conspiracy to dispose of a stolen diamond necklace. The friend jumped bail and cannot be found. Proceeding to trial against the defendant alone, the prosecutor calls the friend’s girlfriend to testify that the friend confided to her that “[the defendant] said I still owe him some of the money from selling that necklace.”

The friend’s girlfriend’s testimony is


A. admissible as evidence of a statement by a party-opponent, the defendant.B. admissible as evidence of a statement against interest by the friend.C. inadmissible, because the friend’s statement was not in furtherance of the conspiracy.D. inadmissible, because the friend is not shown to have firsthand knowledge that the necklace was stolen.

Answer choice B is correct. The general rule for statements against interest made by an unavailable declarant is that a statement qualifies as a hearsay exception if, at the time it was made, it was (i) against the declarant’s pecuniary, proprietary, civil, or penal interest, and (ii) such that a reasonable person would not have made it unless he believed it to be true. The facts show that the statement in this case falls under this hearsay exception, so answer choice B is correct. Answer choice A is incorrect because it is not the defendant’s statement (that the friend owed him money) that is being offered but rather the friend’s statement (which links the defendant to the crime). Answer choice C is incorrect because the statement falls under the statement against interest exception, not the co-conspirator exception. Answer choice D is incorrect because the rule applied in this case does not require the declarant to have firsthand knowledge.

The defendant in a civil fraud case plans to testify regarding the sale of land at issue in the case. The plaintiff seeks, for the sole purpose of impeaching the defendant’s character for truthfulness, to introduce evidence of the defendant’s recent conviction for felony assault. The defendant has filed a motion to exclude evidence of the conviction, and the judge has scheduled a hearing on the defendant’s motion.

Which of the following accurately describes each party’s burden regarding introduction of the conviction?


A. The plaintiff must show that the probative value of the conviction outweighs its prejudicial effect in order to prevail.B. The plaintiff must show that the probative value of the conviction substantially outweighs its prejudicial effect in order to prevail.C. The defendant must show that the probative value of the conviction is outweighed by the prejudicial effect in order to prevail.D. The defendant must show that the probative value of the conviction is substantially outweighed by the prejudicial effect in order to prevail.

Answer choice D is correct. Convictions for felony crimes not involving fraud or dishonesty that are less than 10 years old are admissible to impeach a witness. For a witness other than a criminal defendant, the court may exclude such evidence when the party objecting to the impeachment shows that the probative value is substantially outweighed by the prejudicial effect (i.e., the Rule 403 standard). In this case, a civil defendant seeks to bar introduction of a felony that is less than 10 years old, and thus must meet this standard. Answer choices A and B are incorrect because the plaintiff, as the party seeking to admit the conviction of a non-criminal defendant, does not bear the burden in this case. Rather, the burden is placed on the defendant, as the party seeking to exclude the conviction. Answer choice C is incorrect because it does not accurately describe the defendant’s burden. The defendant must show that the probative value is substantially outweighed by the probative effect.

A defendant is on trial for the brutal murder of a victim. The defendant's first witness testified that in her opinion the defendant is a peaceful and nonviolent person. The prosecution does not cross-examine the witness, who is then excused from further attendance.

Which one of the following is INADMISSIBLE during the prosecution's rebuttal?


A. Testimony by the witness's former employer that the witness submitted a series of false expense vouchers two years ago.B. Testimony by a police officer that the defendant has a long-standing reputation in the community as having a violent temper.C. Testimony by a neighbor that the witness has a long-standing reputation in the community as an untruthful person.D. Testimony by the defendant's former cell mate that he overheard the witness offer to provide favorable testimony if the defendant would pay her $5,000.

Answer choice A is correct. Answer choice A is inadmissible because instances of prior bad acts by a witness may not be proven by extrinsic evidence. Character evidence is any document or testimony offered to prove that a person acted in a particular way on a particular occasion, based on that person's disposition or character. However, evidence of a defendant's bad character is typically inadmissible to prove that the defendant has a propensity to commit crimes and therefore is likely to have committed the crime in question. Answer choice B is incorrect because although the prosecution cannot initially introduce evidence of the defendant's bad character, the defendant made his character an issue in the case when he offered evidence of his good character. When the defendant "opens the door," the prosecution is free to rebut the defendant's claims by attacking the defendant's character with reputation or opinion evidence. Answer choice C is incorrect because the credibility of a witness may be attacked by reputation or opinion evidence as to the witness's character for truthfulness. Answer choice D is incorrect because such testimony is admissible to expose the bias of a witness.

A plaintiff brought an action in civil court against the defendant for battery. The defendant contended that he had acted in self-defense, which depended on the defendant’s knowledge that the plaintiff carried a knife.

Prior to calling the defendant to testify, the defendant’s attorney sought to call as his first witness a former companion of the plaintiff who lived with the plaintiff at the time of the incident. The defendant’s attorney proffered that the companion would testify that, although the companion could not be sure that the plaintiff carried a knife on the day in question, the plaintiff, as he dressed each morning, always snapped a sheath around the lower part of his right leg and inserted a knife into the sheath. The plaintiff’s attorney objected to the introduction of this testimony as irrelevant, since the defendant had not yet introduced evidence that the defendant himself was aware that the plaintiff carried a knife.

May the court admit the companion’s testimony subject to the defendant’s subsequent introduction of such evidence?


A. Yes, because the court may admit otherwise irrelevant evidence when its probative value substantially outweighs the danger of unfair prejudice.B. Yes, because the fact that the plaintiff carried a knife may be conditionally admitted even though its relevance depends on the existence of the defendant’s knowledge of that fact.C. No, because the relevance of the fact that the plaintiff carried a knife depended on the defendant’s knowledge of that fact, which had not been introduced into evidence.D. No, because the companion was not sure that the plaintiff carried a knife on the day in question.

Answer choice B is correct. When the relevance of evidence depends upon whether a fact exists, proof must be introduced sufficient to support a finding that the fact does exist. The court may admit the proposed evidence on the condition that the proof be introduced later. For this reason, answer choice C is incorrect. Answer choice A is an incorrect statement of the law. Irrelevant evidence is inadmissible. Irrelevant evidence would lack probative value. Answer choice D is incorrect because evidence of a person’s habit to prove conduct in accordance with that habit is admissible. Consequently, although the plaintiff’s companion was not sure whether the plaintiff wore a knife on the day in question, the companion could testify as to the plaintiff’s habit to wear a knife.

Why can't I introduce evidence of a gift I made to charity in order to prove good character when I'm a defendant in a criminal fraud case?

When a criminal defendant seeks to prove his good character under Federal Rule of Evidence 404(a)(1), Federal Rule of Evidence 405(a) allows proof only by reputation evidence or opinion evidence, not by specific instances of conduct.

An apartment was burglarized while the owner was away, and a neighbor was charged with the burglary. At the neighbor’s jury trial, the apartment manager sought to testify that an unmanned security camera had captured a video showing the neighbor entering and leaving the apartment just before and after the break-in. The apartment manager further testified that he had viewed the video, but that it had since been accidentally lost. The neighbor, noting personal animosity between the apartment manager and himself, argued that the camera was not working at the time of the break-in and that a video recording was never made.

Which of the following accurately describes the role of the jury in determining issues related to the video?


A. The jury may determine whether the video ever existed, but the judge must determine whether the apartment manager’s testimony accurately reflected the content of the video.B. The jury may determine whether the apartment manager’s testimony accurately reflected the content of the video, but the judge must determine whether the video ever existed.C. The jury may determine both whether the video ever existed and whether the apartment manager’s testimony accurately reflected the content of the video.D. The jury may not determine whether the video ever existed or whether the apartment manager’s testimony accurately reflected the content of the video; these determinations are left to the judge.

Answer choice C is correct. Generally, the trial judge decides preliminary questions as to the admissibility of evidence, while the jury determines the weight and credibility of the evidence, once admitted. With respect to the best evidence rule (which applies when the contents of a video recording are offered to prove the events captured in the recording), the jury determines whether the alleged recording ever existed and, if so, whether other evidence as to the content of that recording reflects that content. Accordingly, answer choices A, B, and D are incorrect because they take all or part of this determination away from the jury.

While riding home on his bike, a cyclist struck his next-door neighbor who was walking across the street. The neighbor suffered serious injuries and sued the cyclist to recover damages stemming from those injuries. The neighbor established, through eyewitness testimony, that the cyclist was travelling at an excessive speed at the time of the accident. On the witness stand, the cyclist was asked by his own attorney about the neighbor’s tendency of carelessly crossing the street without looking. The defense has no other witness to corroborate this tendency.
Which of the following is the neighbor’s best argument to prevent the introduction of cyclist’s testimony as habit evidence?


A. The cyclist’s testimony is self-serving.B. Habit evidence is not admissible when there is an eyewitness to the event.C. The evidence is not sufficiently specific.D. Habit evidence is not admissible because it cannot be corroborated.

Answer choice C is correct. In order for evidence of a person’s conduct to be admissible as habit evidence, the evidence must be sufficiently specific. A tendency to cross the street carelessly is not likely a sufficiently regular response to a particular situation to qualify as a habit; therefore, this is the neighbor’s best argument against admissibility of this evidence. Answer choice A is incorrect because most evidence offered by a party is self-serving. There is no prohibition against such evidence even when the evidence is a party’s own oral testimony. Answer choice B is incorrect because under the Federal Rules of Evidence (the default rules on the MBE), habit evidence is admissible regardless of whether there is an eyewitness to the event. Answer choice D is incorrect because habit evidence need not be corroborated to be admissible.


In a civil trial for fraud arising from a real estate transaction, the defendant claimed not to have been involved in the transaction. The plaintiff called a witness to testify concerning the defendant's involvement in the fraudulent scheme, but to the plaintiff's surprise the witness testified that the defendant was not involved, and denied making any statement to the contrary. The plaintiff now calls a second witness to testify that the first witness had stated, while the two were having a dinner conversation, that the defendant was involved in the fraudulent transaction.

Is the testimony of the second witness admissible?


A. No, because a party cannot impeach the party's own witness.B. No, because it is hearsay not within any exception.C. Yes, but only to impeach the first witness.D. Yes, to impeach the first witness and to prove the defendant's involvement.

Answer choice C is correct. Prior statements that are inconsistent with a witness's present testimony impeach the witness's credibility because they tend to show that the witness's trial testimony is not believable. The prior inconsistent statement was not made under oath, and so does not fit the exemption to the hearsay rule provided by Federal Rule of Evidence 801(d)(1)(A). There is no other hearsay exception that is satisfied under the facts. Therefore, the statement is admissible only to impeach the witness and not for its truth, making answer choice D incorrect. Answer choice A is incorrect because under Federal Rule of Evidence 607, "[t]he credibility of a witness may be attacked by any party, including the party calling the witness." Answer choice B is incorrect because the prior statement of the witness is inadmissible hearsay under Federal Rule of Evidence 802 only if offered to prove that the defendant was involved in the transaction. It is not hearsay if offered to impeach the witness whose trial testimony is inconsistent with it. This is because, whether true or not, the statement is probative to show that the witness is not credible--he said one thing at trial and said something else previously.

A defendant was charged with assault of a stranger. During her trial, the defendant informed her attorney that her friend was with her on the night of the alleged assault and was willing to testify about the details of the event. Coincidentally, the friend was going through a divorce. In her divorce proceedings, the friend testified that she had been with the defendant on the night in question to rebut a claim by her husband that she had been having an affair. Called to the stand in the assault prosecution by the defendant’s attorney, the friend, out of fear that she would also be charged with assault, testified that she was not with the defendant on the night in question.
May the defendant’s attorney introduce into evidence the friend’s prior statement about the night in question as substantive evidence?


A. Yes, because the prior statement is inconsistent with the friend’s current testimony.B. Yes, under the former testimony exception to the hearsay rule.C. No, because the statement is inadmissible hearsay.D. No, because a party may not impeach the party’s own witness.

Answer choice A is correct. A prior inconsistent statement made under penalty of perjury at a trial, hearing, or other proceeding, or in a deposition, may be admissible as substantive evidence. Even a statement made in another action that is unrelated to the current action may be admitted under this rule. Here, the friend’s statement made in the divorce proceeding may be admitted as substantive evidence that she was with the defendant on the night in question. Answer choice B is incorrect because, to qualify under the former testimony exception to the hearsay rule, the declarant must be unavailable to testify. Here, the friend is available to testify, so that exception would not apply. Answer C is incorrect because, although the friend’s testimony at her divorce proceedings was offered for the truth of the matter asserted, it is nonhearsay because it qualifies as a prior inconsistent statement. Answer choice D is incorrect because a party may impeach the party’s own witness.

During a defendant’s civil trial for assault, a witness for the plaintiff testified that, shortly after the assault took place, the defendant had admitted to her that he did assault the plaintiff. On cross-examination, the defendant’s counsel asked the witness whether she had testified in the defendant’s prior criminal prosecution for assault that she had never spoken to the defendant before. The plaintiff objected to the defendant’s question.

May the court allow the question over the plaintiff’s objection?


A. Yes, but only to prove that the defendant did not commit the assault.B. Yes, but only to impeach the credibility of the witness.C. Yes, both to prove that the defendant did not commit the assault and to impeach the credibility of the witness.D. No, because the defendant did not first allow the witness the opportunity to explain or deny the statement.

Answer choice C is correct. A witness’s prior inconsistent statement is always admissible to impeach the witness. Such a statement is also admissible substantively as non-hearsay if the statement was made under oath, as this one was at the prior criminal proceeding. Answer choices A and B are incorrect because they each only recognize one of the two admissible uses for the evidence here. Answer choice D is incorrect because, while the witness must be given the chance to explain or deny the inconsistent statement, the opportunity to explain or deny need not take place before the statement is admitted into evidence.

A defendant, who was charged with the crime of assaulting a victim, admitted striking the victim but claimed to have acted in self-defense when he was attacked by the victim, who was drunk and belligerent after a football game. The defendant offered testimony of his employer that the employer had known and employed the defendant for twelve years and knew the defendant's reputation among the people with whom he lived and worked to be that of a peaceful, law-abiding, nonviolent person.

The trial judge should rule this testimony


A. admissible, because it is relevant to show the improbability of the defendant's having committed an unprovoked assault.B. admissible, because it is relevant to a determination of the extent of punishment if the defendant is convicted.C. not admissible, because whether the defendant is normally a person of good character is irrelevant to the specific charge.D. not admissible, because it is irrelevant without a showing that the employer was one of the persons among whom the defendant lived and worked.

Answer choice A is correct. Character evidence of a criminal defendant is always permitted to show evidence of his good character being inconsistent with the type of crime charged. Here, the employer's testimony is relevant to show the defendant did not commit the crime. Thus, answer choice D is incorrect. Answer choice B is incorrect because the evidence goes to the guilt or innocence of the defendant, not to the separate decision of sentencing. Answer choice C is incorrect because testimony that the defendant is peaceful and law-abiding is directly relevant to refuting the assault charge brought against him.

A cyclist sued a defendant corporation for injuries sustained when she was hit by a truck owned by the defendant and driven by its employee, who was making deliveries for the defendant. The day after the accident, the employee visited the cyclist in the hospital and said, "I'm sorry for what I did." At trial, the employee testified that he had exercised due care.

Why is the cyclist's testimony relating what the defendant's employee said at the hospital admissible to prove negligence?


A. It is a prior inconsistent statement.B. It is a statement against interest.C. It is a statement by a party-opponent's agent.D. It is a statement of then-existing state of mind.

Answer choice C is correct. Federal Rule of Evidence 801(d)(2)(D) provides that a statement is "not hearsay" if it is offered against a party and is "a statement by the party's agent or servant concerning a matter within the scope of the agency or employment, made during the existence of the relationship." Answer choice A is incorrect. It is true that the statement is admissible to impeach as a prior inconsistent statement. Being sorry "for what I did" is arguably inconsistent with the testimony by the employee at trial that he had exercised due care. But a statement admissible only to impeach cannot be used for substantive purposes such as to prove negligence. The statement is admissible as an admission by an agent under Federal Rule of Evidence 801(d)(2)(D). Answer choice B is incorrect. A statement against interest is only admissible if the declarant is unavailable. Here the declarant is available and testifying at trial. Answer choice D is incorrect. The statement "I'm sorry" reflects the employee's state of mind at the time the statement was made. But the relevant issue is not the employee's state of mind the day after the accident but whether he was negligent at the time of the accident. The hearsay exception for state-of-mind statements is not backward looking. It generally does not allow statements of current mental feeling to prove past conduct or to show why the declarant has a certain state of mind.

A businessman was the target of a grand jury investigation into the alleged bribery of American and foreign officials in connection with an international construction project. The businessman had stated at a press conference that no bribes had been offered or taken and that no laws of any kind had been broken. The grand jury issued a subpoena requiring the businessman to testify before it. The businessman moved to quash the subpoena on the ground that his testimony could tend to incriminate him. The prosecutor responded with a grant of use immunity (under which the businessman's compelled statements before the grand jury could not be used against him in any state or federal prosecution). The businessman responded that the grant of use immunity was not sufficient to protect his Fifth Amendment rights.
Should the businessman be compelled to testify?


A. No, because the businessman remains subject to the risk of foreign prosecution.B. No, because use immunity does not prevent the government from prosecuting the businessman on the bribery scheme.C. Yes, because the businessman has denied any criminal liability and therefore his Fifth Amendment rights are not at stake.D. Yes, because the grant of use immunity is coextensive with the businessman's Fifth Amendment rights.

Answer choice D is correct. The U.S. Supreme Court has held that if a person is guaranteed, through a grant of use immunity, that neither his statements nor the fruits of those statements can be used against him in a domestic prosecution, then he loses his right to refuse to testify because his statements cannot tend to incriminate him. Because the businessman has been protected against the use of his statements in a domestic prosecution, his statements cannot tend to incriminate him in any sense protected by the Fifth Amendment. Answer choice A is incorrect. While the businessman does face the possibility of foreign prosecution, the U.S. Supreme Court has squarely held that the Fifth Amendment does not protect against the risk of foreign prosecution.

Cars driven by the plaintiff and the defendant collided, and the defendant was charged with driving while intoxicated in connection with the accident. She pleaded guilty and was merely fined, although under the statute the court could have sentenced her to two years in prison. Thereafter, the plaintiff, alleging that the defendant's intoxication had caused the collision, sued the defendant for damages. At trial, the plaintiff offers the properly authenticated record of the defendant's conviction.

The record should be


A. admitted as proof of the defendant's character.B. admitted as proof of the defendant's intoxication.C. excluded, because the conviction was not the result of a trial.D. excluded, because it is hearsay, not within any exception.

nswer choice B is correct. The document is clearly hearsay - an out-of-court statement offered to prove the truth of the matter asserted - as it is being offered to prove that the defendant was convicted of driving while intoxicated. However, under a hearsay exception, evidence of a felony conviction is admissible in a subsequent trial against the person convicted in order to prove any fact essential to sustain the judgment. Thus, answer choice D is incorrect. Answer choice A is incorrect because the evidence is admissible because it goes to the defendant's negligence through his intoxication, not his general character. Answer choice C is incorrect because Federal Rule of Evidence 803 allows both convictions based on a court or jury verdict as well as convictions based on guilty pleas. Note, however, that pleas of no contest (i.e., nolo contendere) cannot be used in a subsequent trial.

A plaintiff sued a defendant for copyright infringement for using in the defendant's book some slightly disguised house plans on which the plaintiff held the copyright. The plaintiff is prepared to testify that he heard the defendant's executive assistant for copyright matters say that the defendant had obtained an advance copy of the plans from the plaintiff's office manager.

The plaintiff's testimony is


A. admissible as reporting a statement of an employee of a party-opponent.B. admissible as a statement of a co-conspirator.C. inadmissible, because it is hearsay not within any exception.D. inadmissible, because there is no showing that the assistant was authorized to speak for the defendant.

Correct: Answer choice A is correct. A prior out-of-court statement made by a party to the current litigation that is offered against that party is not hearsay. In this case, the defendant's assistant's statement is a vicarious admission, because it is a statement made by an agent or employee during the course of employment and concerning a matter within the scope of employment. Answer choices C and D are incorrect because the hearsay exception applies. Answer choice B is incorrect because there is no indication that there was a conspiracy between the assistant and the defendant.

A plaintiff used aluminum brackets in her business. On the telephone listed as hers in the telephone book, the plaintiff received a call in which the caller said, "This is [the defendant] of [the defendant's hardware company]. We have a special on aluminum brackets this week at 30 percent off." The plaintiff ordered brackets from the caller. When the brackets were never delivered, the plaintiff sued the defendant for breach of contract. At trial, the defendant, who denies having made the telephone call, objects to the plaintiff's testimony concerning it. When asked, the plaintiff testifies that, aside from the telephone call, she had never heard the defendant speak until she met him in the judge's chambers before the trial and that, in her opinion, the voice on the telephone was the defendant's.

The strongest argument for admission of the plaintiff's testimony concerning the telephone call is that


A. the call related to business reasonably transacted over the telephone.B. the call was received at a number assigned to the plaintiff by the telephone company.C. after hearing the defendant speak in chambers, the plaintiff recognized the defendant's voice as that of the person on the telephone.D. self-identification is sufficient authentication of a telephone call.

Answer choice C is correct. Oral statements need to be authenticated as to the identity of the speaker in cases where that identity is important. Here, a statement by the defendant would be an admission by a party-opponent, and the plaintiff's best argument for admission would be to authenticate the call by recognizing his voice. A voice can be identified by any person who has heard the voice at any time, including one made familiar solely for the purposes of litigation. Answer choice A is incorrect because the mere fact that the call related to business does not identify the defendant. Answer choice B is incorrect because it was the defendant who called the plaintiff, not vice versa, and the plaintiff's phone number is not at issue. Answer choice D is incorrect because it misstates the rule, as it applies to situations where an outgoing call is placed.

A plaintiff sued a defendant for breach of contract. The court admitted testimony by the plaintiff that the defendant and his wife quarreled frequently, a fact of no consequence to the lawsuit. The defendant seeks to testify in response that he and his wife never quarreled.

The court


A. must permit the defendant to answer if he had objected to the plaintiff's testimony.B. may permit the defendant to answer, whether or not he had objected to the plaintiff's testimony.C. may permit the defendant to answer only if he had objected to the plaintiff's testimony.D. cannot permit the defendant to answer, whether or not he had objected to the plaintiff's testimony.

Answer choice B is correct. Evidence, whether introduced through testimony or otherwise, must be relevant. Relevant evidence is that which has any tendency to make a fact of consequence more or less probable than it would be without the evidence. Evidence may be admissible even if the risk of prejudice outweighs the probable value--the test is whether the prejudicial value substantially outweighs the probative value. Answer choices A and D are incorrect because the court has the discretion to allow the defendant to respond to the plaintiff's testimony. Answer choice C is incorrect because the court has this discretion regardless of whether or not the defendant first objects to the testimony.


Re-direct examination of a witness must be permitted in which of the following circumstances?


A. To reply to any matter raised in cross-examination.B. Only to reply to significant new matter raised in cross-examination.C. Only to reiterate the essential elements of the case.D. Only to supply significant information inadvertently omitted on direct examination.


Incorrect: Answer choice B is correct. Re-direct is generally limited only to reply to significant new matters raised in cross-examination. Answer choice A is incorrect because it ignores this rule. Answer choice C is incorrect for the same reason; re-direct is only allowed when significant new information arises during cross-examination. Answer choice D is incorrect because such information will be beyond the scope of the original cross-examination.-

In a jurisdiction without a Dead Man’s Statute, the plaintiff’s estate sued the defendant, claiming that the defendant had borrowed from the plaintiff $10,000, which had not been repaid as of the plaintiff’s death. The plaintiff was run over by a truck. At the accident scene, while dying from massive injuries, the plaintiff told a police officer to “make sure my estate collects the $10,000 I loaned to the defendant.”
The police officer’s testimony about the plaintiff’s statement is


A. Inadmissible, because it is more unfairly prejudicial than probative.B. Inadmissible, because it is hearsay not within any exception.C. Admissible as an excited utterance.D. Admissible as a statement under belief of impending death.

Answer choice B is correct. The plaintiff’s statement is hearsay: it was made out of court and is being offered to prove the truth of the matter asserted—that the plaintiff loaned the defendant $10,000. The excited utterance exception does not apply because the hearsay statement does not relate to a startling event or condition; therefore, answer choice C is incorrect. The dying declaration exception does not apply because the statement does not pertain to the cause or circumstances of the plaintiff’s death; therefore, answer choice D is incorrect. Answer choice A is incorrect because the plaintiff’s statement goes directly to the issue of the case, and as such is more probative than prejudicial.
The foregoing NCBE MBE question has been modified to reflect current NCBE stylistic approaches; the NCBE has not reviewed or endorsed this modification.

Answer choice B is correct. The plaintiff’s statement is hearsay: it was made out of court and is being offered to prove the truth of the matter asserted—that the plaintiff loaned the defendant $10,000. The excited utterance exception does not apply because the hearsay statement does not relate to a startling event or condition; therefore, answer choice C is incorrect. The dying declaration exception does not apply because the statement does not pertain to the cause or circumstances of the plaintiff’s death; therefore, answer choice D is incorrect. Answer choice A is incorrect because the plaintiff’s statement goes directly to the issue of the case, and as such is more probative than prejudicial.
The foregoing NCBE MBE question has been modified to reflect current NCBE stylistic approaches; the NCBE has not reviewed or endorsed this modification.

Answer choice B is correct. The eyewitness's statement is hearsay within the hearsay report. The report itself could be admissible as a business or public record, but the hearsay within it is admissible only if it satisfies a separate hearsay exception or if it can be shown that the eyewitness had a business or public duty to report the information accurately. The eyewitness had no such duty. The eyewitness's statement is also not a present sense impression, because it was made the day after the fight, and no other hearsay exception applies. Answer choice A is incorrect. The statement is inadmissible hearsay even if the entire report is introduced; the eyewitness's statement is hearsay within the hearsay report. Answer choice C is incorrect. The fact that the eyewitness purports to have personal knowledge does not solve the hearsay problem, which arises because the eyewitness might not have told the truth about the event he purportedly saw and is not subject to cross-examination about it. Answer choice D is incorrect. The eyewitness's statement is hearsay within the hearsay report. The report itself could be admissible as a business or public record, but the hearsay within it is admissible only if it satisfies a separate hearsay exception or if it can be shown that the eyewitness had a business or public duty to report the information accurately. The eyewitness had no such duty, and no other hearsay exception applies.

I


n a prosecution of a defendant for assault, a witness is called to testify that the victim had complained to the witness that the defendant was the assailant.

The witness's testimony is most likely to be admitted if the witness is


A. a doctor, whom the victim consulted for treatment.B. a minister, whom the victim consulted for counseling.C. the victim's husband, whom she telephoned immediately after the event.D. a police officer, whom the victim called on instructions from her husband.

Answer choice C is correct. The most persuasive answer is that the victim's statement was an excited utterance: a statement made about a startling event or condition while the declarant is under the stress of excitement caused by the event. To be an excited utterance, the event must shock or excite the declarant, and the statement must relate to the event, but the declarant need not be a participant in the event. The key issue is not that the victim called her husband, but rather that she placed the call immediately after the event. Answer choices A, B, and D are less persuasive because they are hearsay without an exception, and are thus incorrect. Answer choice A does not fall under the hearsay exception for statements for purposes of medical diagnosis or treatment because that exception applies only to statements that describe the declarant's medical history or past or present symptoms, pain, or other sensation: it does not apply to a declarant identification of an assailant. Answer choices B and D are incorrect because there are no special hearsay exceptions for statements made to ministers or police officers.

A defendant was charged with attempted murder of a victim in a sniping incident in which the defendant allegedly shot at the victim from ambush as the victim drove his car along an expressway. The prosecutor offers evidence that seven years earlier, the defendant had fired a shotgun into a woman’s home and that the defendant had once pointed a handgun at another driver while driving on the street.
How should the court rule on the admissibility of this evidence?


A. Exclude it, because such evidence can be elicited only during cross-examination.B. Exclude it, because it is improper character evidence.C. Admit it as evidence of the defendant’s propensity toward violence.D. Admit it as relevant evidence of the defendant’s identity, plan, or motive.

Answer choice B is correct. Evidence of a defendant’s past crimes or wrongful acts is inadmissible if it is offered to prove that the defendant has a propensity to commit crimes and therefore is likely to have committed the crime in question. Accordingly, answer choice C is incorrect. Answer choice A is incorrect because, regardless of whether on cross or direct, character evidence may not be offered to prove conduct in conformity with that character trait. Answer choice D is incorrect because, although prior bad acts may be offered under the MIMIC rule, the defendant’s two previous bad acts are so unrelated to the case at hand that they do not tend to prove identity, plan, or motive.
The foregoing NCBE MBE question has been modified to reflect current NCBE stylistic approaches; the NCBE has not reviewed or endorsed this modification.

Several defendants, senior executives of a corporation, were charged with securities fraud. The government called as a witness another executive of the corporation, who had not been charged and who had been given immunity from prosecution, to authenticate handwritten notes that she had made after meetings of the corporation’s management team at which the alleged fraud was discussed. The witness testified that she had prepared the notes on her own initiative to help her remember what had happened at the meetings. After this testimony, the government offered the notes into evidence to establish what had happened at the meetings.

Should the witness’s notes be admitted?


A. No, because the notes are hearsay not within any exception.B. No, because the witness’s immunity agreement with the government makes her notes untrustworthy and thus substantially more prejudicial than probative.C. Yes, because they are business records.D. Yes, because they are past recollections recorded.

A.

In a suit based on a will, inheritance of $1 million depended upon whether the wife had survived her husband when both died in the crash of a small airplane. An applicable statute provided that, for purposes of distributing an estate after a common disaster, there was a rebuttable presumption that neither spouse had survived the other. A witness was called to testify that as she approached the plane she heard what she thought was a woman's voice saying, "I'm dying," although by the time the two occupants were removed from the wreckage they were both dead.

Is the witness's testimony admissible?


A. No, because the matter is governed by the presumption that neither spouse survived the other.B. No, because the witness's testimony is too speculative to support a finding.C. Yes, because the hearsay rule does not apply to statements by decedents in actions to determine rights under a will.D. Yes, because it is relevant and not otherwise prohibited.

Answer choice D is correct. The testimony is not barred by the hearsay rule or any other rule and is relevant on the issue of whether the wife survived the husband. The majority common law rule, which is followed by Rule 301 of the Federal Rules of Evidence, adopts the "bursting bubble" view under which a presumption disappears when sufficient counterproof is offered about the presumed fact. Here the testimony of the witness, although not conclusive, is sufficient to rebut the presumption that neither spouse survived the other and to support a jury finding that the wife outlived the husband. Therefore, the presumption is no longer controlling, and the witness's testimony is admissible; as such, answer choice A is incorrect. Answer choice B is incorrect because the testimony is not "too speculative." A witness need not be absolutely certain of matters about which the witness testifies. Answer choice C is incorrect because the statement by the woman is not hearsay, because it is not being offered to prove the truth of the matter asserted. It is being offered to prove that the woman was alive at the time she made the statement.

A defendant, charged with armed robbery of a store, denied that he was the person who had robbed the store. In presenting the state's case, the prosecutor seeks to introduce evidence that the defendant had robbed two other stores in the past year.

This evidence is


A. admissible, to prove a pertinent trait of the defendant's character and the defendant's action in conformity therewith.B. admissible, to prove the defendant's intent and identity.C. inadmissible, because character must be proved by reputation or opinion and may not be proved by specific acts.D. inadmissible, because its probative value is substantially outweighed by the danger of unfair prejudice.

Answer choice D is correct. Here, evidence of the defendant's prior bad acts is not admissible to show his criminal propensity because the defendant has not opened the door by offering evidence of his good character. Thus, answer choice A is incorrect. Under the MIMIC rule, a defendant's past crimes or other wrongful acts may be admissible as circumstantial evidence of motive, intent, absence of mistake, identity, or common plan or scheme. Under these facts, however, the MIMIC exception does not apply; the danger of undue prejudice is substantially greater than the probative value of the evidence. Thus, answer choice B is incorrect. Answer choice C is incorrect, because it misstates the law: once the defendant has put his character in issue, the prosecution, on cross-examination, may question a character witness about specific instances of the defendant's conduct.